Q18

User avatar
 
Crogati
Thanks Received: 2
Jackie Chiles
Jackie Chiles
 
Posts: 32
Joined: January 12th, 2013
 
 
 

Q18

by Crogati Sun Feb 03, 2013 1:00 pm

Might someone let me know if I chose the right answer and advise on the approach I used?

So, if Quigley arrives at some time before the Rockville, then the Wooster must arrive (c) fourth. Justified by the hypothetical:
Q R T/V W S T/V Y

I eliminated (A) because if W is placed in spot 2, then Y must be in 4, which places S in 3 (W-S-Y sequence) and leaves no room for the other sequences.

I eliminated (B) (D) (E) because W could not be placed in the given spots without violating the W-S-Y sequence.

My approach to this question was to eliminate the answers based on W's placement. Is that an OK method or do you suggest something better?

I am also confused about how I classified the wrong answers. Since this is a "must be" question the strategy should be to find the "must be" answer, and eliminate the "could be" wrong answers, but I found the wrong answers are "must be false."

Is this the rare occurrence about which the strategy guide admonished: when the wrong answers for a "must be true" question occasionally are "must be false?"
User avatar
 
ohthatpatrick
Thanks Received: 3805
Atticus Finch
Atticus Finch
 
Posts: 4661
Joined: April 01st, 2011
 
This post thanked 1 time.
 
 

Re: Q18

by ohthatpatrick Wed Feb 06, 2013 3:00 am

You are correct: the way this question is worded, one answer must be true, the other four must be false.

That will generally happen if each answer choice is a number.

For instance, in an "In-Out" type game, you often get asked something like, "What is the maximum number of things in the IN column?"

There's only 1 number that correctly answers the question, so the correct answer must be true and the other four must be false.

I don't think that really changes our strategy/approach too much.

For this question, I would initially symbolize
Q - R
and then attach any other rules that deal with Q or R
Q - R - T,V
and then consider the other rules
W - S - Y

These two chains account for all 7 people. The next big choice is whether W or Y will be spot 4. I would probably draw a scenario in which W is 4 and one in which Y is 4.

Once we discover, as you did, that Y can't be 4, we know W must be.

Good work!
User avatar
 
Crogati
Thanks Received: 2
Jackie Chiles
Jackie Chiles
 
Posts: 32
Joined: January 12th, 2013
 
 
 

Re: Q18

by Crogati Sun Feb 10, 2013 7:10 pm

Thanks for explaining! :D :D